2/3 ( 12 n + 6 ) − 1 5 ( 10 n − 2 )

Answers

Answer 1

Answer:

34-158n

Step-by-step explanation:

First factor:

8n+4-150n+30

Then combine like terms

34-158n

Answer 2

Answer:    -142n+34

Step-by-step explanation:(2/3)(12n)+(2/3)(6)+(-15)(10n)+(-15)(-2)

=-142n+34


Related Questions

Park rangers at a game preserve in Africa observed a cheetah run 3

3

5

miles in

1

20

hour. What was the cheetah’s speed in miles per hour (mph)?

A.

9

50

mph

B. 5

5

9

mph

C. 72 mph

D. 92 mph

2

Among other ingredients, a recipe for paella calls for

3

4

pound of chorizo and

1

2

cups of rice. What is the unit rate of pounds of chorizo to cups of rice in the recipe?

A.

3

10

pound of chorizo per cup of rice

B.

8

15

pound of chorizo per cup of rice

C. 1

7

8

pound of chorizo per cup of rice

D. 3

1

3

pound of chorizo per cup of rice

3

Mary Ellen has a white oak tree in her backyard. She observed that the tree had grown

2

3

foot in

3

4

year. What is the tree’s growth rate in feet p

Answers

Answer:

Cheetah's speed = 2.79 miles per hour

Step-by-step explanation:

Speed = Distance / Time

Distance = 335 miles, Time = 120 hours

So, speed = 335 / 120

Speed = 2.79 mph

A count went from 605 to 203. What was the approximate percent decrease? Round the numbers to find an estimate of the percent decrease.

Answers

Answer:

66 %

Step-by-step explanation:

it is 66% because 203 is less than half of 605 .So, you would add together .

203+203=406   Thats not even close to 605 so now you add almost half so that would be about 66%

IT'S D

HOPE THIS HELPED MARK ME BRAINLIST PLS

Can you guys answer 1 and 2?

Answers

the picture is black or maybe it’s just me sorry

help pls thank you!!

Answers

10/2.5= 4
2/3*4= 8/3

2 2/3 is your answer (C)

Answer:

C 2+2/3

Step-by-step explanation:

10/2.5= 4

2/3*4= 2+2/3

Chicken costs $3.20 per pound, and beef costs $4.59 per pound. Answer each question and show your reasoning. When purchasing 3 pounds, which meat is more expensive? By how much more expensive is it?

Answers

Given :

Chicken costs $3.20 per pound, and beef costs $4.59 per pound.

To Find :

When purchasing 3 pounds, which meat is more expensive and by how much.

Solution :

Price of 3 pound chicken, [tex]P_c=3.2\times 3 = \$9.6[/tex] .

Price of 3 pound beef, [tex]P_b = 4.59\times 3=\$13.77[/tex] .

Now, difference in price :

P = $13.77 - $9.6 = $4.17 .

Therefore, beef is more expensive than chicken by $4.17 .

Hence, this is the required solution.

Given that sec(0) =25/7
and 0 is in Quadrant I, find csc(0) and tan(0).

Answers

Answer:

Csc(0)=24/27 and Tan(0)= 24/7

Step-by-step explanation:

We know that secant is the inverse of cos. So that Sec(x) = Cos^-1(x)= hypotenuse/adjacent. We now know that 25 is hypotenuse and 7 is the length of side adjacent to angle theta.

Given this information we can find the length of side opposite to theta, which has length 24.

Now we know:

Hypotenuse=27

Adjacent=7

Opposite=24

Csc is inverse of sin, so that scs=sin^-1=hypotenuse/opposite=27/24

and tan=opposite/adjacent=24/7

What 10 x 13x 5 x6 x8 x9 x9 x0

Answers

Answer:

0

Step-by-step explanation:

If those x's are NOT variables, then the answer is 0. The x is a multiplication sign, right?

To solve the following system by elimination of the x terms, if the first equation were multiplied by -4, by what number would you multiply the second equation?

Answers

Answer:

6

Step-by-step explanation:

Multiply by 6 and you can eliminate x or y.

I've worked out the problem multiple times but I don't get any of the answer choices. Can someone work it and tell me what you get? The answer I have selected is closest to what I'm getting.

Answers

♥️♥️♥️♥️♥️♥️♥️♥️♥️♥️♥️♥️♥️♥️

[tex] \frac{ {a}^{3} - 27 }{ {a}^{2} - 9 } \div \frac{ {a}^{2} + 3a + 9 }{a + 3} = \\ [/tex]

[tex] \frac{ {a}^{3} - 27 }{ {a}^{2} - 9 } \times \frac{a + 3}{ {a}^{2} + 3a + 9 } = \\ [/tex]

[tex] \frac{(a - 3)( {a}^{2} + 3a + 9) }{(a - 3)(a + 3)} \times \frac{a + 3}{ {a}^{2} + 3a + 9 } = \\ [/tex]

[tex] \frac{(a - 3)(a + 3)( {a}^{2} + 3a + 9) }{(a - 3)(a + 3)( {a}^{2} + 3a + 9) } = \\ [/tex]

[tex]1[/tex]

♥️♥️♥️♥️♥️♥️♥️♥️♥️♥️♥️♥️♥️♥️

The correct answer is the first option .

♥️♥️♥️♥️♥️♥️♥️♥️♥️♥️♥️♥️♥️♥️

Please anyone help me

Answers

Answer: Are we supposed to solve for y or?

Step-by-step explanation:

warning: sorry if this doesn't make sense but this all i can get out of this one

-2x + y = -9

x = 9/2 + y/2

y = -9 + 2x

------

same warning here

12x + 3y = 9

x =3/4 - y/2

y = 3 - 4x

sorry if this doesn't help but i hope it does

18 + 19x = 10x + 54
can someone help me with this

Answers

Answer:

The answer is X= 4.

Write the equation of the line that passes through the given points.

Answers

Answer:

y=3x-3

Step-by-step explanation:

the y(intercept)= -3 because the point is at -3 on the y-axis. from there the line goes up 3 and over 1. the slope would be 3.

All together the equation of the line is:

Y=3x-3

Brainliest to right answer, dont answer if your not sure...also based on the previous answer are the equations equivalent?

Answers

Answer:

D

Step-by-step explanation:

To get from A to B you must divide by x on each side, therefore dividing by the same variable expression

i need help with a question! im going to post it now! some one please help!!!!
its math

Answers

OKAYYYYYYYYYYYYYYYYYYYYYYYY

Answer:

Where is it?

Step-by-step explanation:

La profesora repartió témperas del mismo tamaño entre los chicos. Mariana usó 3/5 de su pote y Juan 3/4 del suyo. ¿Quién usó mayor cantidad y por qué? Representa con dibujos los potes y lo que se usó

Answers

Answer:

Juan uso mayor cantidad de témpera.

Step-by-step explanation:

Para comparar fracciones que tienen distintos denominadores, como en este caso, es conveniente encontrar fracciones equivalente a las fracciones dadas, donde ambas tengan el mismo denominador. Dos fracciones son fracciones equivalentes cuando representan la misma parte de la unidad.

En este caso, para comparar las fracciones 3/5 y 3/4, obtienes que el mínimo común múltiplo entre los denominadores que es 20 (el denominador común se puede obtener multiplicando ambos denominadores o calculando el m.c.m) y amplificas cada una de las fracciones para que tengan el mismo denominador.

Entonces:

[tex]\frac{3}{5} =\frac{3*4}{5*4} =\frac{12}{20}[/tex]

[tex]\frac{3}{4} =\frac{3*5}{4*5} =\frac{15}{20}[/tex]

Cuando dos o más fracciones tienen igual denominador, es mayor la que tiene mayor numerador. Entonces en este caso podes observar 15/20 es mayor. Como 3/4 es una fracción equivalente a 15/20, entonces 3/3 es una fracción mayor. Por lo que es posible decir que Juan uso mayor cantidad de témpera.

Esto también se puede observar en la representación de la imagen adjunta, donde se divide en 1/4 y 1/5 y se encuentra pintado la cantidad usada por Juan y Mariana. En ese caso se puede ver que el área pintada es mayor en el caso de 3/4, la cantidad usada por Juan.

Solve the literal equation

X= (a+b+c)/(ab)

for a.

Answers

Answer:

a = (b + c)/(bx - 1)

Step-by-step explanation:

Given

x= (a+b+c)/(ab)

Solving for a

x = (a + b+ c)/(ab)abx = a + b+ cabx - a = b + ca(bx - 1) = b + ca = (b + c)/(bx - 1)

- 6
48
-
99
how to solve it​

Answers

Huh? Can you put a picture?

I need help on this... i have to find the m

Answers

Answer:

100°

Step-by-step explanation:

(20x)° = (7x + 5)° + 60°

20x - 7x = 65

13x = 65

x = 5

m∠BTU = (20 × 5)° = 100°

Consider the expression shown.


(x^-3 y^5 z^-4) • (x^6 y^-7 z^-2)


Part A. The value of the exponent of x in the simplified expression will be


1. -18


2. -9


3. 3


Part B. The value of the exponent of y in the simplified expression will be


1. -35


2. -12


3. -2


Part C. The value of the exponent of z in the simplified expression will be


1. -6


2. -2


3. 8

Answers

Answer:

3, 3 and 1

Step-by-step explanation:

Using the rule of exponents

[tex]a^{m}[/tex] × [tex]a^{n}[/tex] = [tex]a^{(m+n)}[/tex]

Given

([tex]x^{-3}[/tex] [tex]y^{5}[/tex] [tex]z^{-4}[/tex] × [tex]x^{6}[/tex] [tex]y^{-7}[/tex] [tex]z^{-2}[/tex]

A

[tex]x^{-3}[/tex] × [tex]x^{6}[/tex] = [tex]x^{(-3+6)}[/tex] = x³ → with exponent 3

B

[tex]y^{5}[/tex] × [tex]y^{-7}[/tex] = [tex]y^{(5-7)}[/tex] = [tex]y^{-2}[/tex] → with exponent - 2

C

[tex]z^{-4}[/tex] × [tex]z^{-2}[/tex] = [tex]z^{(-4-2)}[/tex] = [tex]z^{-6}[/tex] → with exponent - 6

At an office supply store, 8 boxes of pens and 3 folders cost $6.50. At the same store, 8 boxes of pens and 1 folder cost $5.90. How much does one box of pens cost? How much does 1 folder cost? Please explain.

Answers

Answer:

$0.30

Step-by-step explanation:

the difference of the folders is 2.

The difference of money is $0.60

60/2 = 30

$0.30

Kinsley can illustrate 9 story book pages in 3 hours. How many pages can she
illustrate per hour?
Hint: Make a W.O.N. table to solve.
o A) 27 pages
B) 9 pages
o C) 1 page
D) 3 pages

Answers

Answer:

3 pages

Step-by-step explanation:

because if she reads 9 pages in 3 hours if you multiply each answer by 3 you should be able to find the right answer

Imad's family drove 345 miles from Chicago to Cleveland.
They drove at a speed of 50 miles per hour. How many hours did it take Imad's family to drive from Chicago to Cleveland?
A
7.1 hr
B
6.35 hr
6.9 hr
D
7.9 hr

Answers

Answer:

6.9 is the answer

Step-by-step explanation:

345 divided by 55. hope this helps....

Find y if 3х + 4y = 1

Answers

Answer:

y=1/4-3/4y

Step-by-step explanation:

rearrange the equation

4y=1-3x

divide both sides by 4 to get the value of y

Answer:

y = 1/4

Step-by-step explanation

Find Y set x=0

3(0)+4y = 1

        4y = 1

         y  = 1/4

2x+2y=38
Y=x+3

Substitute to solve

Correct answer gets Brainliest

Answers

Answer:

x=8

y=11

Step-by-step explanation:

2(8) + 2(11) = 38

8 + 3 = 11

If two pounds of steak will serve 5 people, how many
pounds will be needed to serve 13 people?

Answers

Answer:

5.2 pounds will be needed

Step-by-step explanation:

TEXAS INSTRUMENTS TI-30X IX

- calculator i used

Answer:

5.2 approximately 5 pounds of steak

Step-by-step explanation:

if 5 people : 2 pounds of steak

then 13 people : ?

if more , less divides

13/5×2

26/5

5.2

4. The difference log2 (3) - log2 (12) is equal to

Answers

Answer:

Step-by-step explanation:

-2

Apply this particular rule of logs:

log a - log b = log a/b

Therefore, log2 (3) - log2 (12) = log2 (3/12) = log2 (1/4)

This last result simplifies to  log2 1 - log2 4 = 0 - 2 = -2

Answer:

-2

Step-by-step explanation:

[tex]log_2(3)-log_2(12)[/tex]

~Simplify using log properties

[tex]log_2(\frac{3}{12})[/tex]

[tex]log_2(\frac{1}{4})[/tex]

~Apply log rules

[tex]-log_2(4)[/tex]

~Rewrite

[tex]-log_2(2^2)[/tex]

~Apply log rules

[tex]-2log_2(2)[/tex]

~Apply log rules

-2 * 1

~Simplify

-2

Best of Luck!

4. A map has a scale of 1 inch = 100 miles. The distance between two cities is 7.25 inches. If a car travels
50 miles per hour, about how long will it take to get from one city to another?
hours

Answers

Answer:

14.5 hours/14 hours 30 minutes

Step-by-step explanation:

The distance between the two cities is 7.25•100=725 because you multiply the scale distance by the scale.The number hours is 725/50=14.5 hours because you divide the distance by 50 (the miles per hour).

ILL GIVE Brainliest
find the area of the polygon

Answers

Answer:

27.5 units^2

Step-by-step explanation:

Total area = Area of triangle ILH + Area of parallelogram JKLI

Area of triangle ILH

= 1/2 * b * h

= 1/2 * 5 * 5

= 12.5 units^2

Area of parallelogram JKLI

= bh

= 3 * 5

= 15 units^2

Total area

= Area of triangle ILH + Area of parallelogram JKLI

= 12.5 + 15

= 27.5 units^2

Paul's bank account used to have $1,250 dollars in it. After saving and following a strict budget he manages to increase that amount by 40%. How much money is in his bank account now?

Answers

Answer: $1750

Step-by-step: see the picture

Please mark me brainliest!! :)

Find f (h(-5))
f (x) 9x - 5
g(x) - 3x
h(x) 2x2

Your answer:​

Answers

hi

You are making  f ° h  

f(x) = 9x-5

h(x) = 2x²  

You must first calculate   h(-5)   :     so remplace  x  by -5 in h :

h(-5) = 2 (-5)² =  2* 25 = 50

then apply  f(x) to result  :   f(50) = 9 *50 -5 = 450 -5 = 445  

Other Questions
How do I do an energy profile diagram What determines an organisms traits at the molecular scale? What two methods are the best choices to factor this expression 18x2-8 A pair of shoes was $5 but is now on sale for$4. What is the percent discount? How was Nestor 10 found? This model shows how cold winter air is warmed in the Great Lakes Basin, which creates ideal temperatures for year-roundfruit farming, which statement best describes this interaction?A)B)The biosphere and hydrosphere interact, which affects theatmosphereThe geosphere and atmosphere interact, which affects thehydrosphere.The hydrosphere and atmosphere interact, which affects thebiosphere.The geosphere and hydrosphere interact, which affects theatmosphereC)D) A cold water faucet can fill the bathtub in 12 minutes, and a hot water faucet can fill the bathtub in 18 minutes. The drain can empty the bathtub in 24 minutes. If both faucets are on and the drain is open, how long would it take to fill the bathtub?The answer is not 10.3, I dunno why Read the following excerpt from John Muir's article "The Wild Parks and Forest Reservations of the West." Then answer the question that follows.Only thirty years ago, the great Central Valley of California, five hundred miles long and fifty miles wide, was one bed of golden and purple flowers. Now it is ploughed and pastured out of existence, gone forever--scarce a memory of it left in fence corners and along the bluffs of the streams. The gardens of the Sierra, also, and the noble forests in both the reserved and unreserved portions are sadly hacked and trampled, notwithstanding, the ruggedness of the topography--all excepting those of the parks guarded by a few soldiers. In the noblest forests of the world, the ground, once divinely beautiful, is desolate and repulsive, like a face ravaged by disease. This is true also of many other Pacific Coast and Rocky Mountain valleys and forests. The same fate, sooner or later, is awaiting them all, unless awakening public opinion comes forward to stop it.How does Muir try to persuade his audience in this excerpt?He encourages readers to get as angry as he is about the loss of natural areas.He appeals to readers' sense of guilt about building cities within wilderness areas.He encourages readers to stop trampling the land so that it remains beautiful.He appeals to readers' feelings of nostalgia for natural beauty that might be lost. Water is made of three atoms. This means it is The medieval thinker that influenced Scholasticism was _______ Please I need help I give brainliest!! Parler vous francais 1. Simplify by using the distributive property: 4 (2x +9) Do you believe that congressional members should be elected by popular vote? Why or why not? Please answer this!! Please answer this :(Explain your reason for the least important and most important contribution. (Short answer: 2 to 3 sentences) PLEASE HELP WILL GIVE BRAINLIEST Help ASAP plz help for 10 point Does knowing the origin of velocity help you understand the difference between speed and velocity? Charlie bought a pair of shorts at the store when they were having a 45% off sale. If the regular price was 24$ how much did charlie pay?